Lesson: Weaken the Argument Questions

Comment on Weaken the Argument Questions

gmat-admin's picture

Yes, Assumptions often take a lot of time to answer.

I'm not quite sure what you're asking when you ask "Can I just think only one assumption or just skip it?"
If you're asking how many Assumption questions you can expect on test day, I'd say you'll see 1 or 2 of them.

Cheers,
Brent

so sorry to make you confuse. I meant "an unstated assumption."
as you mentioned in the lesson, we should come up with these things from the argument.
-premises
-an unstated assumption.(***I meant this part)
-conclusion

So I have a hard time to come up with "unstated assumption", if I just have one that would be an issue?
gmat-admin's picture

I totally agree that it's often quite hard to identify possible assumptions. I think students shouldn't spend more than 10 or 15 seconds trying to determine unstated assumptions in an Assumption questions.

Hey Brent, I have two questions:

I kind of feel very comfortable without taking notes, I feel like I quite well grasp the cloud of intention evolving around a passage very well. Still, sometimes I do feel that it would have helped me to take notes (especially identifying clearly the conclusion). Since I read a lot about taking, I try to do it, but it takes more time and I don´t necessaryily feel that it increases my answer choice probability. Do you have any suggestions? I would like to hear your opinion on the situation.

And one questions that despoite the Guide´s explanation is kind of not so clear to me:

Most of the year, the hermit thrush, a North American songbird, eats a diet consisting mainly of insects, but in autumn, as the thrushes migrate to their Central and South American wintering grounds, they feed almost exclusively on wild berries. Wild berries, however, are not as rich in calories as insects, yet thrushes need to consume plenty of calories in order to complete their migration. One possible explanation is that berries contain other nutrients that thrushes need for migration and that insects lack.

Which of the following, if true, most seriously calls into question the explanation given for the thrush’s diet during migration?


(A) Hermit thrushes, if undernourished, are unable to complete their autumn migration before the onset of winter.

(B) Insect species contain certain nutrients that are not found in wild berries.

(C-correct) For songbirds, catching insects requires the expenditure of significantly more calories than eating wild berries does.

(D-Why not correct?) Along the hermit thrushes’ migration routes, insects are abundant throughout the migration season.

(E) There are some species of wild berries that hermit thrushes generally do not eat, even though these berry species are exceptionally rich in calories.

Cheers,

Philipp
gmat-admin's picture

When it comes to taking or not taking notes, everyone is different. Some students are super strong readers with great memories, and others read more slowly and need to take notes.

To determine the approach that's best for you, I suggest you perform an experiment in which you test two different strategies (taking notes and not taking notes). If you measure your success rates and timing for each approach, you'll have a good idea of which approach is best for you.

Question link: https://gmatclub.com/forum/most-of-the-year-the-hermit-thrush-a-north-am...

Summary: Even though insects provide more energy (i.e., calories) than berries, the thrush choose to eat berries during their migration.
Why would they eat berries when insects provide more energy?
EXPLANATION: berries contain other nutrients that thrushes need for migration and that insects lack.

We want a new premise that weakens the explanation.

(C) For songbirds, catching insects requires the expenditure of significantly more calories than eating wild berries does.
This provides an alternative explanation as to why the thrush eat berries during migration: Catching insects takes too much energy. That is, the caloric benefits from insects are offset because the thrush have to work much harder to catch insects.

(D-Why not correct?) Along the hermit thrushes’ migration routes, insects are abundant throughout the migration season.
This does not weaken the explanation that thrushes eat berries BECAUSE they contain special nutrient for migration.
If anything this answer choices strengthens the explanation by noting that the thrush eat the berries even when insects are abundant during that time.

Answer: C

Does that help?

Cheers,
Brent

Now I understand the reasoning behind, thanks!

Hi Brent,

I have just started going through the GMAT prep course you have prepared and want to thank you for your effort. I also like it that for each CR question type you have compiled a list of questions to practice. However, I would like to make a suggestion regarding that.

Can you turn the bulleted list into a numbered list instead? I know that's a minor thing to ask of but sometimes I kind of lose track of the question I am working on and end up opening the link to the previously attempted questions. With a numbered list, it would be easier to track what questions we have attempted and we can move to the next question.

Just a small suggestion, which I hope might be useful for others as well. Hope that won't be too much of an effort for you.

Once again, thank you for the insightful videos.
gmat-admin's picture

Hi Anamika,

That's a great idea. Unfortunately, it wouldn't necessarily solve the issue, because I often add questions to various parts of each list (i.e., I add some easier questions at the top of the list, some medium question to the middle of the list, and so on).
So, adding practice questions to the list would change the numbering of the questions, so you wouldn't know what was covered.

HOWEVER, other students have asked for the same thing. So, my programmer made it so that, each time you click a link, the link changes color. So, when you return to the list of questions, you can see which ones you've already answered.
If, for some reason, the links aren't changing color for you, you may need to change a setting in your Web browser.

I hope that helps.

Cheers,
Brent

Hi Brent,

Could you please explain why B is incorrect and A is correct:

79. Increased use of incineration is sometimes advocated as a safe way to dispose of chemical waste. But opponents of incineration point to the 40 incidents involving unexpected releases of dangerous chemical agents that were reported just last year at two existing incinerators commissioned to destroy a quantity of chemical waste material. Since designs for proposed new incinerators include no additional means of preventing such releases, leaks will only become more prevalent if use of incineration increases.

Which of the following, if true, most seriously weakens the argument?

(A) At the two incinerators at which leaks were reported, staff had had only cursory training on the proper procedures for incinerating chemical waste.

(B) Other means of disposing of chemical waste, such as chemical neutralization processes, have not been proven safer than incineration.

(C) The capacity of existing incinerators is sufficient to allow for increased incineration of chemical waste without any need for new incinerators.

(D) The frequency of reports of unexpected releases of chemical agents at newly built incinerators is about the same as the frequency at older incinerators.

(E) ln only three of the reported incidents of unexpected chemical leaks did the releases extend outside the property on which the incinerators were located.

Thank you in advance,
gmat-admin's picture

The conclusion is that we should expect more and more leaks.
The implied conclusion is that the FAULTY DESIGN caused the releases of dangerous chemical agents.

Answer choice A suggests that POOR STAFF TRAINING may have caused the releases of dangerous chemical agents.
So, with better training, the incineration process may be safer, in which case, we may see fewer leaks in the future.
This weakens the argument.

Answer choice B says that other processes are even worse than the incineration process.
So, it seems we're stuck with incineration (since it appears to be the best option).
There's nothing here that weakens the argument that we should expect more and more leaks

Does that help?

Cheers,
Brent

Hi Brent,

Thank you very much,

I think if this was a strengthening the argument question then A would be a correct choice,

Am I right?
gmat-admin's picture

Did you mean to say A?
Answer choice A clearly weakens the argument.
If this were a strengthen the argument question, I would say that answer choice D would be best because it suggests that new incinerators are just as faulty as old incinerators.

Sorry, I meant B,

I think in terms of strengthening B is much stronger than D, it amplifies that incarceration is the best option, while D suggests that it is basically on the same level.

What do you think?
gmat-admin's picture

The conclusion of the argument is that leaks will become more prevalent if the use of incineration increases.

So, if we're trying to strengthen that conclusion then answer choice D tells us that the frequency of leeks will increase when more incinerators are built.

Answer choice B just tells us that incineration maybe the safest route. This however does not strengthen the conclusion that leaks will become more prevalent.

Ok,

Thank you a lot Brent

Hi Brent,

Could you please why GMAT creators put the "B" as an answer choice?

Wind farms, which generate electricity using arrays of thousands of wind-powered turbines, require vast expanses of open land. County X and County Y have similar terrain, but the population density of County X is significantly higher than that of County Y. Therefore, a wind farm proposed for one of the two counties should be built in County Y rather than in County X.

Which of the following, if true, most seriously weakens the planner’s argument?


(A) County X and County Y are adjacent to each other, and both are located in the windiest area of the state.

(B) The total population of County Y is substantially greater than that of County X.

(C) Some of the electricity generated by wind farms in County Y would be purchased by users outside the county.

(D) Wind farms require more land per unit of electricity generated than does any other type of electrical-generation facility.

(E) Nearly all of County X’s population is concentrated in a small part of the county, while County Y’s population is spread evenly throughout the country.


In my opinion, B contradicts completely with the statement of the argument.

We are told that X and Y have similar terrains, so their area is about the same. Then we are told that the population density of X is greater than in Y, which would mean that X has more people than Y since their areas are the same. Therefore, "The total population of County Y is substantially greater than that of County X" is at odds with what we know. If we take B as true then it is the weakening of the argument. What has higher power, the question itself or the answer options?

Thank you in advance,
gmat-admin's picture

I found two instances up this question online (https://gmatclub.com/forum/wind-farms-which-generate-electricity-using-a... and https://gmatclub.com/forum/wind-farms-which-generate-electricity-using-a...)

In both cases, the official answer is given as E

Cheers,
Brent

Hi Brent,

I understand,

But my entire question was about B being at odds with the statement itself, and how to approach it? If we consider B as a valid statement, then it also weakens the argument(by saying that it provides incorrect information)

"In my opinion, B contradicts completely with the statement of the argument.

We are told that X and Y have similar terrains, so their area is about the same. Then we are told that the population density of X is greater than in Y, which would mean that X has more people than Y since their areas are the same. Therefore, "The total population of County Y is substantially greater than that of County X" is at odds with what we know. If we take B as true then it is the weakening of the argument. What has higher power, the question itself or the answer options?"

Thank you in advance,
gmat-admin's picture

Sorry. I originally read your question as suggesting the GMAT test-makers are saying that the correct answer is B.

The main problem with your rationale is that we are not told that counties X and Y have the same AREA. All we are told is that they have similar TERRAINS. Terrain refers to the physical features of land (e.g., mountainous, flat, rolling hills, etc).

Thank you Brent,

I understand what you mean,

Hi Brent,

Could you please explain why A is a better choice than E,

In my oppinion there is nothing said about the loop: worse infrostructure→ fewer people→ worse infrastructure→fewer people

Urban planner: When a city loses population due to migration, property taxes in that city tend to rise. This is because there are then fewer residents paying to maintain an infrastructure that was designed to support more people. Rising property taxes, in turn, drive more residents away, compounding the problem. Since the city of Stonebridge is starting to lose population, the city government should therefore refrain from raising property taxes.

Which of the following, if true, would most weaken the urban planner's argument?

(A) If Stonebridge does not raise taxes on its residents to maintain its infrastructure, the city will become much less attractive to live in as that infrastructure decays.

(B) Stonebridge at present benefits from grants provided by the national government to help maintain certain parts of its infrastructure.

(C) If there is a small increase in property taxes in Stonebridge and a slightly larger proportion of total revenue than at present is allocated to infrastructure maintenance, the funding will be adequate for that purpose.

(D) Demographers project that the population of a region that includes Stonebridge will start to increase substantially within the next several years.

(E) The property taxes in Stonebridge are significantly lower than those in many larger cities.

What do you think?

Thank you in advance,
gmat-admin's picture

Question link: https://gmatclub.com/forum/urban-planner-when-a-city-loses-population-du...

The implied conclusion here is that we should NOT raise taxes because doing so will make Stonebridge even less attractive, which will cause even more people to leave.

In the passage, we learn of the following cycle:
People leave (due to migration) → Taxes increase → People leave (due to higher taxes) → Taxes increase.

We can generalize the cycle as:
People leave (due to some undesirable feature of the city) → Taxes increase → People leave (due to some undesirable feature of the city)

The proposal to NOT raise taxes is based on the idea that raising taxes will create an undesirable feature of the city that will cause people to leave.

Answer choice A says that, by NOT raising taxes, we create a different undesirable feature: the city's infrastructure will decay. In other words, by not paying taxes, we create another (yet different) undesirable feature of the city.
This definitely weakens the argument.

(E) does not necessarily weaken the argument that we should not raise taxes. Regardless of the tax rate (whether lower or higher than other cities), Stonebridge still needs a certain amount of money to run a successful city, and when people leave, the city collects fewer taxes.

Cheers,
Brent

Hi Brent,

Thank you very much,

Very helpful,

Hi Brent,

Could you please explain why E is incorrect and why B is correct;

E is probably incorrect because the argument focuses on Vidnet's shows and not on all networks combined. Nevertheless, I can't substantiate why B is a correct answer.

The more viewers a television show attracts, the greater the advertising revenue the show generates. The television network Vidnet's most popular show, Starlight, currently earns the network's highest profits, but next year, because of unavoidable increases in production costs, its profits are projected to fall to below the average for Vidnet shows. Therefore, Vidnet would earn greater profits overall if it replaced Starlight with a show of average popularity and production costs.

Which of the following, if true, most seriously weakens the argument?

(A) The average profits of Vidnet shows have increased in each of the last three years.

(B) Shows that occupy time slots immediately before and after very popular shows tend to have far more viewers that they otherwise would.

(C) Starlight currently has the highest production costs of all Vidnet shows.

(D) Last year Vidnet lost money on a weekly show that was substantially similar to Starlight and was broadcast on a different day from Starlight.

(E) Even if, as a result of increased production costs, Starlight becomes less profitable than the average for Vidnet shows, it will still be more profitable than the average for television shows of all networks combined.


Thank you in advance,
gmat-admin's picture

Question link: https://gmatclub.com/forum/the-more-viewers-a-television-show-attracts-t...

PREMISE: More viewers = more advertising revenue
PREMISE: Next year, Starlight's profits will be less than average
CONCLUSION: Replace Starlight (with average show) and earn greater profits

Answer choice B tells us that, in addition to generating revenue on their own, popular shows also help generate revenue for the time slots before and after most popular shows.
So, although Vidnet will see higher profits in Starlight's previous time slot, the shows before and after Starlight's previous time slot will see fewer viewers, which means the advertising revenue from those 2 shows will decrease.

So, overall, Vidnet could actually lose money by replacing Starlight.
This weakens the argument that replacing Starlight an average show will help increase profits.

Yes,

I understand what you mean.

Great explanation, thank you Brent

Hi Brent,

Could you please explain why B is better answers than A.
B- doesn't explain why there was an increase in the number of turtles returning to the beach
A - also doesn't explain why there was an increase in the number of turtles returning to the beach

A major chemical spill occurred five years ago at Baker’s Beach, the world’s sole nesting ground for Merrick sea turtles, and prevented nearly all the eggs laid that year from hatching. Yet the number of adult female Merricks returning to lay their eggs at Baker’s Beach has actually increased somewhat since five years ago. Clearly, environmentalists’ prediction that the world’s Merrick population would decline as a result of the spill has proven unfounded.

Which of the following, if true, most seriously undermines the argument offered in refutation of the environmentalists’ prediction?


A. The chemical spill five years ago occurred at a time when there were neither Merrick sea turtles nor Merrick sea turtle eggs on Baker’s Beach.

B. Female Merrick sea turtles begin returning to Baker’s Beach to lay their eggs when they are ten years old.

C. Under normal conditions, only a small proportion of hatchling female Merrick sea turtles survive in the ocean until adulthood and return to lay their eggs at Baker’s Beach.

D. Environmental pressures unrelated to the chemical spill have caused a significant decline in the population of one of the several species of sea birds that prey on Merrick sea turtle eggs.

E. After the chemical spill, an environmental group rejected a proposal to increase the Merrick sea turtle population by transferring eggs from Baker’s Beach to nearby beaches that had not been affected by the spill.
gmat-admin's picture

Question link: https://gmatclub.com/forum/a-major-chemical-spill-occurred-five-years-ag...

CONCLUSION: The prediction that the spill would CAUSE a decrease in the Merrick population was unfounded.

We're told that "nearly all the eggs laid that year did not hatch"
Since this is a premise, we must assume that it is true.

Answer choice A seems to contradict this premise.
If there were no turtles or eggs on the beach when the spill occurred, how could the spill prevent the eggs from hatching?
One possible explanation is that the spill occurred BEFORE the turtles came to lay their eggs.
However, since the spill may have contaminated the beach, it's possible that nearly all the eggs laid didn't hatch.

What really matters here is the conclusion.
Does answer choice A weaken the conclusion that "the prediction that the spill would CAUSE a decrease in the Merrick population was unfounded"?
No. It's still quite possible that the spill caused a decrease in the population.
After all, the spill did occur the same year that the eggs did not hatch.

Now let's take a closer look at answer choice B.
It tells us that the turtles, once hatched, don't return to Baker's Beach until they are 10 years old.
It is presently 2019.
This means that the turtles returning this year were born in 2009
And the turtles that returned last year were born in 2008
etc.

So, if the chemical spill happened in 2014 (and prevented the eggs from hatching that year), then we won't see the repercussions until 2024, and the females born in 2014 will become 10 years old and return to Baker's Beach to lay eggs.

So, up until 2024 all returning turtles were born BEFORE the spill occurred.
Given this, it is entirely reasonable for the returning populations to have increased for the past 5 years.

This weekend's the conclusion that the prediction that the spill would CAUSE a decrease in the Merrick population was unfounded.

Does that help?

Hi Brent,

Do I need to write down the premise and conclusion as I read the argument. Can I avoid this step as I find it time consuming? Please help. Thanks.
gmat-admin's picture

I think some students benefit from physically writing the premises and conclusion while others experience no advantage with that strategy. So, it's up to you.

Hi Sir,

Simply put, for any weaken questions, can I just find the assumption and then weaken it?

Thank you!
gmat-admin's picture

Yes, that will work. The hard part is that an argument can have countless assumptions.

Hey Brent!

Can you please help with this question? The correct answer is 'e' but doesn't this disprove the premise that 'the lawyers who advertise a specific service usually charge less for that service than lawyers who do not advertise'?

The fewer restrictions there are on the advertising of legal services, the more lawyers there are who advertise their services, and the lawyers who advertise a specific service usually charge less for that service than lawyers who do not advertise. Therefore, if the state removes any of its current restrictions, such as the one against advertisements that do not specify fee arrangements, overall consumer legal costs will be lower than if the state retains its current restrictions.

Which of the following, if true, would most seriously weaken the argument concerning overall consumer legal costs?


(A) The state has recently removed some other restrictions that had limited the advertising of legal services.

(B) The state is unlikely to remove all of the restrictions that apply solely to the advertising of legal services.

(C) Lawyers who do not advertise generally provide legal services of the same quality as those provided by lawyers who do advertise.

(D) Most lawyers who now specify fee arrangements in their advertisements would continue to do so even if the specification were not required.

(E) Most lawyers who advertise specific services do not lower their fees for those services when they begin to advertise.
gmat-admin's picture

Question link: https://gmatclub.com/forum/the-fewer-restrictions-there-are-on-the-adver...

P: Fewer rules means more lawyer ads
P: Advertising lawyers usually work for cheap
C: Fewer rules means lower overall legal costs

The implied assumption here is that lawyers who don't work for cheap will do both of the following:
1) start advertising
2) lower their fees

However, it may just be the case that, with fewer rules, more and more of the cheap lawyers will begin advertising (and keep their already-cheap fees the same), and the non-cheap lawyers will choose not to advertise and continue charging their regular fees. In this scenario, more lawyers are advertising, but no one is charging less than they previously charged. This seriously weakens the argument, and it's basically what answer choice E describes.

Does that help?

Yulia's picture

Hi Brent,

Could you please advise how to approach such a question? https://gmatclub.com/forum/assembly-hansen-my-opponent-in-this-election-campaign-has-accused-me-219624.html
gmat-admin's picture

When it comes to GMAT Verbal questions, it's best to stick with Official questions (there are 1000's of official questions).

I'm not a big fan of this question.
One could easily argue that the primary conclusion is that Assembly Hansen has no right to make accusations (given her history). If we accept this as the primary conclusion, then answer Choice D is irrelevant.
Conversely, if we say that the primary conclusion is that the author is not guilty of inappropriately using public money, then Choice D is great.

Yulia's picture

Hi Brent,
Could you please break down the question, I cant understand the correct choice?

https://gmatclub.com/forum/editorial-in-ledland-unemployed-adults-receive-government-256350.html

Editorial: In Ledland, unemployed adults receive government assistance. To reduce unemployment, the government proposes to supplement the income of those who accept jobs that pay less than government assistance, thus enabling employers to hire workers cheaply. However, the supplement will not raise any worker's income above what government assistance would provide if he or she were not gainfully employed. Therefore, unemployed people will have no financial incentive to accept jobs that would entitle them to the supplement.

Which of the following, if true about Ledland, most seriously weakens the argument of the editorial?

(A) The government collects no taxes on assistance it provides to unemployed individuals and their families.

(B) Neighboring countries with laws that mandate the minimum wage an employer must pay an employee have higher unemployment rates than Ledland currently has.

(C) At any given time, people who are currently employed have the best chance of being offered a job that will give them an income significantly greater than government assistance would give them.

(D) The financial assistance that the government provides to people who have no other income is less than the average starting wage.

(E) People sometimes choose a job for reasons that have nothing to do with the financial benefits it offers.

Hi Brent,

https://gmatclub.com/forum/in-colorado-subalpine-meadows-nonnative-dandelions-co-occur-with-a-na-201645.html

In this really tough question, the correct answer choice E requires us to think so deeply into the possibilities of the option.

I am actually struggling with such hard questions, I am not able to connect the vagueness to how it could possibly impact the answer.

I hope you are able to understand the issue.

Thanks.

Hi Brent,

could you please help answer this question?

https://gmatclub.com/forum/mayor-the-financial-livelihood-of-our-downtown-businesses-is-in-jeopa-321898.html

Thanks
gmat-admin's picture

Thank you, it's very clear to me now.

Hi Brent. Good Afternoon.
In 'Reinforcement Activities' question#9 - 'Highway Official: When resurfacing our concrete bridges...', I am confused between option 'A' & 'D'. Can you please explain how 'D' doesn't fit in as a solution?
gmat-admin's picture

Question link: https://gmatclub.com/forum/highway-official-when-resurfacing-our-concret...

The highway official argues that ECC is great for bridges since the electricity needed to de-ice bridges ends up costing less using salt.
Actual cost to surface the bridge with ECC is much more expensive than not using ECC.
As such, the taxpayer is against ECC (on economic grounds).

The question asks, "Which of the following, if true, could best be used to support the highway official’s proposal in the face of taxpayer’s objection?"
The taxpayer's objection is all about money.
So the correct answer choice will be the one that addresses the economic aspects of the proposal to use ECC on the bridge.

(A) The use of de-icing salt causes corrosion of the reinforcing steel in concrete bridge decks and damage to the concrete itself, thereby considerably shortening the useful life of concrete bridges.
In other words, salt (the alternative to ECC) significantly damages concrete bridges, which will cost taxpayers a lot of money. Since ECC eliminates the need for salting the bridges, the life of the bridges are increased. So, ECC makes sense from an economic standpoint.

(D) Aside from its potential use for de-icing bridges, ECC might also be an effective means of keeping other concrete structures such as parking garages and airport runways ice free.
Answer choice D does not address the question.
That is, it doesn't address the taxpayer's objection about ECC costing too much to install.

Does that help?

Thanks, it really helps.

Hi Brent. Good Evening.
In 'Reinforcement Activities' question#21 - 'The price the government pays for standard weapons...', I am confused between options 'A' & 'E'. Can you please explain how 'E' doesn't fit in as a solution?
gmat-admin's picture

Question link: https://gmatclub.com/forum/the-price-the-government-pays-for-standard-we...

Hi Arjita,
In the future, please post a link to the question. It's much easier for me to find the question that way.

We have to pay close attention to what the question is asking us. It asks:
Which of the following statements, if true, is the best basis for a criticism of historical costing as an ECONOMICALLY SOUND pricing method for military contracts?

Answer choice A addresses the economical soundness of the pricing method. It points out that, if a deal between the government and a contractor was a bad deal for the government, then the pricing method would lock the government into continuing with that bad deal.

Answer choice E, on the other hand, tells us that the pricing method doesn't encourage the development of innovative weapons.
Since answer choice E doesn't address the economical soundness of the pricing method, we can eliminate it.

Thanks, it really helps.

Hi Brent. Good Afternoon.
In 'Reinforcement Activities' question#23 - 'Archaeologist: Researchers excavating a burial site in Cyprus...', I am confused between option 'A' & 'D'. Can you please explain how 'A' doesn't fit in as a solution?

Pages

Office Hours

On December 20, 2023, Brent will stop offering office hours. 

Change Playback Speed

You have the option of watching the videos at various speeds (25% faster, 50% faster, etc). To change the playback speed, click the settings icon on the right side of the video status bar.

Have a question about this video?

Post your question in the Comment section below, and a GMAT expert will answer it as fast as humanly possible.

Free “Question of the Day” emails!